PDA

View Full Version : Monk: What's wrong, and what can be done?



Tola
2007-03-17, 11:27 AM
I've heard it around here that there are problems with this class.

The title really says it all.

(Actually, while we're at it, we should say what's 'wrong' with other classes as well.)

Yuki Akuma
2007-03-17, 11:44 AM
What's wrong with monks? Many things. Inconsistancy of abilities, for example; they have a high movement rate, and yet their signature special attack requires them to stay still and use a full attack action.

They also need to spend more money than a normal fighter on enchanting their main weapons, or they have to fight with a different weapon and lose another of their class abilities (improved damage dice on unarmed strikes, this time).

Their signature special attack, Flurry of Blows, actually makes them do less damage statistically, until they reach about ninth level, when they start to actually hit with the damn thing.

Their special abilities seem to be mainly special defenses, which don't help out in a party situation at all. This is why most people say the monk's thing is surviving and dragging his friends off to a church to get raised.

Clementx
2007-03-17, 11:47 AM
That and being an incredible pain to spellcasters. They can single-move Tumble through all your wizard's minions, resist and save against all their interdicting spells, and start stunning them, and fighting nearly equally well if you throw a Dispel at them. They are great urban/slightly-tamed-wilderness scouts as well.

Yuki Akuma
2007-03-17, 11:55 AM
That and being an incredible pain to spellcasters.

No. They're really not.

Tola
2007-03-17, 11:56 AM
To ClementX: Aren't those GOOD things?

To Yuki: Explain, please? NEVER just say something like: 'they're not' without explaining. It saves people asking the obvious 'Why?'. Like now, for instance.

Yuki Akuma
2007-03-17, 11:58 AM
To ClementX: Aren't those GOOD things?

It's actually a bad thing. People assume the monk would be good at that, then find out he really isn't very good at grappling (the main reason people seem to think monks are good against spellcasters, for some reason), realise that there are spells that don't allow saving throws or spell resistance that can really hurt them... and, oh yeah, that spellcaster is flying and monks don't have any effective ranged attacks.

There's also the whole MAD thing: if the monk wants good saves, he needs high Dexterity, Constitution and Wisdom. If he wants a good AC, he needs good Dexterity and Wisdom. If he wants a chance to actually hit anything, he needs high Strength, or high Dexterity and a wasted feat slot. If he wants to do any sort of damage with his unarmed strikes, he needs a high Strength (and Power Attack doesn't even work, because his fists are Light weapons).

He can't possibly do everything a monk is "meant to do" without some obscenely lucky rolls or something like 40 point buy.

The Great Skenardo
2007-03-17, 12:00 PM
No. They're really not.

I have to disagree. For your standard squishy wizard or sorceror, the thing he hates the most is to have someone close with him in the first round (someone, it might be added, who has an exellent Touch AC) as well as the best progression for saves in the game and who naturally gains spell resistance later on, as well as evasion. Additionally, no spellcaster likes that same person to get into his fries with a grapple attack (perhaps even with Improved Grapple as a bonus feat). This effect can be compounded if the Monk takes the feat Mage Slayer (but that's just extra.)

Your more combat capable Divine casters might not be outplussed by this as much, but a Monk can also make their lives difficult with Improved Disarm (making Divine Might far less useful) or Improved Trip (both of these are potentially bonus feats)
A druid would probably be the best primary caster, against a monk, but the others can be severely hampered by a Monk's unique ability to get into your fries quickly and keep you from casting the spells you want to cast.

EDIT for successive posts: True, Monks aren't always the best at grappling, mostly due to their subpar BAB and the fact that STR is usually a tertiary stat for them. however, it remains quite competetive with any primary spellcaster of comparable level, seeing as most Wiz/Sor/Clr don't invest too much into STR, and equal or lesser BAB progression.
As to flying, yes, that can create a problem, if the spellcaster has had time to prepare, but if the Monk's incredible jumping ability can't put him within reach, I don't think it's unreasonable to assume that the monk might notice that this can happen, so might invest in a magical item or two (even potions of fly, if you don't want wondrous items) to counteract that weakness. I think this is probably feasbile, by the time you face spellcasters capable of flying.
As to Killz Death spells, also true. However, I would point out that these spells spell equal doom for anyone they'er cast against. Monks are strong vs. spellcaster, but not impervious.

Yuki Akuma
2007-03-17, 12:04 PM
I have to disagree. For your standard squishy wizard or sorceror, the thing he hates the most is to have someone close with him in the first round (someone, it might be added, who has an exellent Touch AC) as well as the best progression for saves in the game and who naturally gains spell resistance later on, as well as evasion. Additionally, no spellcaster likes that same person to get into his fries with a grapple attack (perhaps even with Improved Grapple as a bonus feat). This effect can be compounded if the Monk takes the feat Mage Slayer (but that's just extra.)

Your more combat capable Divine casters might not be outplussed by this as much, but a Monk can also make their lives difficult with Improved Disarm (making Divine Might far less useful) or Improved Trip (both of these are potentially bonus feats)
A druid would probably be the best primary caster, against a monk, but the others can be severely hampered by a Monk's unique ability to get into your fries quickly and keep you from casting the spells you want to cast.

If there's a monk standing next to you and you want to cast a spell, you take a step backwards. Monks don't have reach.

And yes, sure, a monk can ready an action to grapple you if you move or try to cast a spell... but he can't do anything else in the round if he does that.

InaVegt
2007-03-17, 12:06 PM
I have to disagree. For your standard squishy wizard or sorceror, the thing he hates the most is to have someone close with him in the first round (someone, it might be added, who has an exellent Touch AC) as well as the best progression for saves in the game and who naturally gains spell resistance later on, as well as evasion. Additionally, no spellcaster likes that same person to get into his fries with a grapple attack (perhaps even with Improved Grapple as a bonus feat). This effect can be compounded if the Monk takes the feat Mage Slayer (but that's just extra.)As yuki already said a decent wizard is both flying and doesn't give the monk a save.


Your more combat capable Divine casters might not be outplussed by this as much, but a Monk can also make their lives difficult with Improved Disarm (making Divine Might far less useful) or Improved Trip (both of these are potentially bonus feats)That's why you have a locked gauntlet as a cleric and are a dwarf.

A druid would probably be the best primary caster, against a monk, but the others can be severely hampered by a Monk's unique ability to get into your fries quickly and keep you from casting the spells you want to cast.Not true, the wizard flies away and the cleric has plenty of options to defend against the monk.

Clementx
2007-03-17, 12:07 PM
If there's a monk standing next to you and you want to cast a spell, you take a step backwards. Monks don't have reach.

Then he readies an action to chuck a shuriken into your face if you cast a spell. Special monk weapon, he just attacks with them out of nowhere.

And don't pretend that all these caster advantages ONLY apply to monks. They apply to any melee-focus class, and are generally required to make up for the wizard's low hit points (arrows are a comparable threat to this HP as would a great ax to a fighter) or cleric's lack of combat style.

martyboy74
2007-03-17, 12:08 PM
blah blah blah
Mage: I take a 5 foot step away from him, and cast Wall of X/Fly/Invisibility/Disintegrate.

Yuki Akuma
2007-03-17, 12:09 PM
Then he readies an action to chuck a shuriken into your face if you cast a spell. Special monk weapon, he just attacks with them out of nowhere.

Oh no. Concentration check DC 11.5 + maybe 1 or 2 from the monk's strength. The absolute horror.


And don't pretend that all these caster advantages ONLY apply to monks. They apply to any melee-focus class, and are generally required to make up for the wizard's low hit points (arrows are a comparable threat to this HP as would a great ax to a fighter) or cleric's lack of combat style.

No, of course not. It's a problem for all melee-oriented classes. But it hurts the monk the most.

Morty
2007-03-17, 12:12 PM
Monks maybe aren't wizard bane, but they're ceratinly better at dealing with wizards than other meleers. Besides, wizards are broken, so that questin is preety pointless in the first place.
For me, monks don't even have to exist; they're completely superfluous. To fix the monk, you have to completely rewrite him. Seriously, can you imagine someone killing a dragon with his fists? Even with all this purity-of-body-and-mind-monastery-training thing.

Yuki Akuma
2007-03-17, 12:13 PM
Monks maybe aren't wizard bane, but they're ceratinly better at dealing with wizards than other meleers. Besides, wizards are broken, so that questin is preety pointless in the first place.
For me, monks don't even have to exist; they're completely superfluous. To fix the monk, you have to completely rewrite him. Seriously, can you imagine someone killing a dragon with his fists?

Maybe if he's a Tiger Claw/Setting Sun swordsage...

Hey, there's a thought. You want to fix monks? Use the unarmed/unarmed swordsage variant in Tome of Battle.

Clementx
2007-03-17, 12:14 PM
Oh no. Concentration check DC 11.5 + maybe 1 or 2 from the monk's strength. The absolute horror.

Plus 1+1d6 shocking or whatever handy enchantment the monk picks up. Nevermind that by time a caster has the Concentration modifier to always make that DC, the monk can afford pretty nasty poisons as well. I have never seen shuriken thrown by a lvl5+ monk in that situation that didn't have a poison on it (one that he makes the save on with a 4 or so, on the 5% chance he nicks himself until lvl11 in which case he doesn't care).

PirateMonk
2007-03-17, 12:14 PM
Seriously, can you imagine someone killing a dragon with his fists? Even with all this purity-of-body-and-mind-monastery-training thing.

Wizards kill dragons by waving their hands at them. :smallamused:

Morty
2007-03-17, 12:15 PM
Maybe if he's a Tiger Claw/Setting Sun swordsage...

Hey, there's a thought. You want to fix monks? Use the unarmed/unarmed swordsage variant in Tome of Battle.

Well, that's actually a good idea. Swordsage is like monk should've been from the very beginning.


Wizards kill dragons by waving their hands at them.

Yeah, this waving causes terrible arcane powers to unleash on dragon. They don't try to kill dragon by beating him to death.

Yuki Akuma
2007-03-17, 12:15 PM
Plus 1+1d6 shocking or whatever handy enchantment the monk picks up. Nevermind that by time a caster has the Concentration modifier to always make that DC, the monk can afford pretty nasty poisons as well. I have never seen shuriken thrown by a lvl5+ monk in that situation that didn't have a poison on it (one that he makes the save on with a 4 or so, on the 5% chance he nicks himself until lvl11 in which case he doesn't care).

So the monk has to waste lots of money of getting ammunition enhanced with damage special abilities and has to waste tons of money on each dose of poison?

Well, sure, if you don't want those Bracers of Armour, or that Cloak of Flying...

(This is also why Vow of Poverty monks aren't very good: they need equipment just as much as everyone else. Just not for the same reasons as the fighter.)

Matthew
2007-03-17, 12:20 PM
If he wants to do any sort of damage with his unarmed strikes, he needs a high Strength (and Power Attack doesn't even work, because his fists are Light weapons).

Yuki, Monks can use the Power Attack Feat with their Fists. Everybody always gets this wrong, but if you read the Power Attack Feat description again, you'll notice the special exemption for Natural and Unarmed Attacks. I am pretty sure this has been mentioned before in other threads about the lameness of Monks and it doesn't help them out, but there you go.

Tola
2007-03-17, 12:23 PM
Why's he bad at grappling? I'm seeing nothing in the grapple section that screams 'Monks are bad at this'...

When does flight come into play? Let's see...Spell Level 3, that's....Level 5, it says here? Can that be right? Neutralise many land-bound creatures so soon?

Bleh. How hard is it to jump and grab a wizard out of the air?

Well. Those are the problems, now..what to do about it...

Let's see...full attack bonus, perhaps?

Giving him flight is not really feasible, I guess....Some sort of major bonus to Jump? Make him not need the run-up?(Again, the grab him out of the air thing...)

Shift the flurry boosting down a few levels?

Would that be too much? Too LITTLE?

Yuki Akuma
2007-03-17, 12:24 PM
Yuki, Monks can use the Power Attack Feat with their Fists. Everybody always gets this wrong, but if you read the Power Attack Feat description again, you'll notice the special exemption for Natural and Unarmed Attacks. I am pretty sure this has been mentioned before in other threads about the lameness of Monks and it doesn't help them out, but there you go.

...Oh. Yeah, I knew that. No mistakes here. Just testing!

:smallredface:

Krellen
2007-03-17, 12:25 PM
The Monk should have a full BAB and the Skirmish ability. Not flurry of blows.

Morty
2007-03-17, 12:25 PM
When does flight come into play? Let's see...Spell Level 3, that's....Level 5, it says here? Can that be right? Neutralise many land-bound creatures so soon?

The whole flight chain of spells was designed when drunk. Monks or any other class doesn't have anything to do with it.
Really, if you want to fiz a monk, use swordsage. I'm generally against replacing standard classes with ToB classes, but in this case it works.

The Great Skenardo
2007-03-17, 12:27 PM
Phew! Okay, a few numbered responses.

1. But TGS, what's keeping the Wizard from taking a 5-ft step as a free action? Monks don't have reach!
I would point out, first of all, that defensive casting makes even that usually unecessary.
Very true, but as I see it, if the monk is attempting to actively harm the wizard, he doesn't end his action right next to the wizard, regardless. I envision hostilities breaking out and (barring Celerity cheese), the Monk will often win Initiative. During this round, he charges the Wizard and initiates a grapple. Against almost any primary arcane caster, a monk of comparable level and decent stats will tend to win. This move is made feasible by the movement speed of the monk, which can cover quite bit of distance as part of a charge. If you fail the grapple, that's when you'd end up next to the spellcaster, and then it might be a problem. ( I would mention in passing that Monks usually have the highest Touch AC of any base class, which makes hitting them with a disintegrate harder, increasing the chance for missing)

2. What about Stability McLovesHisWeapon, the dwarven cleric with the locked gauntlet?
I would argue that this is an exception to the norm. A person who wanted to defend against the possibility of any monk anywhere from depriving him of his precious balance or weapon, then it's certainly possible to do so. (Incidentally, while the cleric Somatic components are often quite simplistic, I don't think you can actually do them whilst wearing a locked gauntlet, neither can you deliver touch attacks thusly.) In that case, grappling would still be a quite viable option (although potentally trickier, as a per the cleric's better strength and BAB). I think most people would agree that this is an extremely atypical cleric build, however.

3. But What about Flying? If the spellcaster flies away, you're kinda screwed, right?
Potentially. first axiom of monks is that you need to be able to reach your target. For the beginning levels of the game, unless you're being pelted from above in a wide open field, a Monk's stellar speed and jumping abilities allow her to reach spellcasters through climbing and jumping. if you''re in a typical dungeon or building (or even in an area with things to climb), then the spellcaster has to be very very careful with her maneuvering, if she wants to simultaneously evade the Monk and still deal with her companions.
Come later levels, the Monk has more options available in the form of Abundant Step and wondrous items bought or purchased along the way. The most versiatile monk is she who can reach any opponent no matter how high she flies.

I would also note that a spellcaster can screw over any class, but short of a Spellthief (who would have the same problems or worse with reaching the spellcaster), the Monk is the base class best suited for shutting down a spellcaster.

Matthew
2007-03-17, 12:29 PM
I would like to see them get Mobile Flurry of Blows, Mobile Two Weapon Fighting, a Full Base Attack Bonus and a Flurry of Blows progression that mirrored Two Weapon Fighting, so that a Monk 20 with Flurry of Blows and Two Weapon Fighting would get:

Standard Attack:

Primary: +16 / +16
Off Hand: +16

Full Attack:

Primary: +16 / +16 / +11 / +11 / +6 / +6 / +1 / +1
Off Hand: +16 / +11 / +6 / +1

Oh yeah, and the ability to get their Full Strength Bonus with Off Hand Attacks...

Yuki Akuma
2007-03-17, 12:33 PM
Why's he bad at grappling? I'm seeing nothing in the grapple section that screams 'Monks are bad at this'...

He's not that bad, but he's not as good as many people think. The fighter is better with his high BaB and lessened MAD.


When does flight come into play? Let's see...Spell Level 3, that's....Level 5, it says here? Can that be right? Neutralise many land-bound creatures so soon?

Alter Self. Spell level two.


Bleh. How hard is it to jump and grab a wizard out of the air?

Can you make a DC 400 jump check?


Let's see...full attack bonus, perhaps?

Sure. Doesn't hurt at all.


Giving him flight is not really feasible, I guess....Some sort of major bonus to Jump? Make him not need the run-up?(Again, the grab him out of the air thing...)

Giving him ToB maneuvers would really sort most of the problems out easily. Balance of the Air allows... well, balancing on the air. Searing Charge gives you a fly speed.


Shift the flurry boosting down a few levels?

Yes. That really needs to be done...

Rahdjan
2007-03-17, 12:33 PM
Play to have fun with a good group of people and none of the classes need reworked. I played a monk whose call to fame WAS his survivability. He took the risks the other players refused to take and always came out on top. Monks just fit the non-role of the group. Don't depend on them for anything specific and they will do amazing things.

P.S. I do like the swordsage idea

Piccamo
2007-03-17, 12:36 PM
Phew! Okay, a few numbered responses.

1. But TGS, what's keeping the Wizard from taking a 5-ft step as a free action? Monks don't have reach!
I would point out, first of all, that defensive casting makes even that usually unecessary.
Very true, but as I see it, if the monk is attempting to actively harm the wizard, he doesn't end his action right next to the wizard, regardless. I envision hostilities breaking out and (barring Celerity cheese), the Monk will often win Initiative. During this round, he charges the Wizard and initiates a grapple. Against almost any primary arcane caster, a monk of comparable level and decent stats will tend to win. This move is made feasible by the movement speed of the monk, which can cover quite bit of distance as part of a charge. If you fail the grapple, that's when you'd end up next to the spellcaster, and then it might be a problem. ( I would mention in passing that Monks usually have the highest Touch AC of any base class, which makes hitting them with a disintegrate harder, increasing the chance for missing)

Even without Celerity spells like Nerveskitter (level 1 spell, immediate action) grant a bonus to Initiative.


2. What about Stability McLovesHisWeapon, the dwarven cleric with the locked gauntlet?
I would argue that this is an exception to the norm. A person who wanted to defend against the possibility of any monk anywhere from depriving him of his precious balance or weapon, then it's certainly possible to do so. (Incidentally, while the cleric Somatic components are often quite simplistic, I don't think you can actually do them whilst wearing a locked gauntlet, neither can you deliver touch attacks thusly.) In that case, grappling would still be a quite viable option (although potentally trickier, as a per the cleric's better strength and BAB). I think most people would agree that this is an extremely atypical cleric build, however.
Pretty much all my melee characters have a locked gauntlet if their primary weapon can be disarmed. You have no significant advantage over the cleric for grapple checks and absolutely none over the druid.


3. But What about Flying? If the spellcaster flies away, you're kinda screwed, right?
Potentially. first axiom of monks is that you need to be able to reach your target. For the beginning levels of the game, unless you're being pelted from above in a wide open field, a Monk's stellar speed and jumping abilities allow her to reach spellcasters through climbing and jumping. if you''re in a typical dungeon or building (or even in an area with things to climb), then the spellcaster has to be very very careful with her maneuvering, if she wants to simultaneously evade the Monk and still deal with her companions.
Come later levels, the Monk has more options available in the form of Abundant Step and wondrous items bought or purchased along the way. The most versiatile monk is she who can reach any opponent no matter how high she flies.

Ranged weapons don't matter since they can be completely negated by a Wind Wall. Abundant step gets you to the caster and then waiting until next turn to act.


I would also note that a spellcaster can screw over any class, but short of a Spellthief (who would have the same problems or worse with reaching the spellcaster), the Monk is the base class best suited for shutting down a spellcaster.
Monks are worse off because they can't do the damage to bring down a caster and they really don't do much else to affect combat.

The Great Skenardo
2007-03-17, 12:38 PM
One further note about flight. If the Monk is already in your fries you can certainly take a 5-foot step and cast fly, but you can't thereafter fly away. You have a move action left, but something people often forget is that you can't move in a round that you take a five foot step. If you could, the withdrawal action wouldn't exist.

EDIT:

1. All my melee guys have a locked guantlet, and divine primary spellcasters are much harder to grapple.
I think it's a little odd that locked guantlets are the default. They restrict your ability to do anything in combat but swing your weapon. you can't brandish your holy symbol to turn undead, you can't drink a potion, use a scroll or wand, or even use any spell that requires material components, unless you forgo having a shield.
Druids aren't meant to be grappled. In that case, you're better off trying to trip them up before they transform into a bear. I admit, in the face of a druid with natural spell, there's not a whole not a monk can do. On the other hand, it's similarly difficult for the druid to affect you with any of her spells (as druid spells with no save/no spell resistance tend to be far more sparse than arcane spells)
Clerics are harder, but still feasbile with the +4 bonus to Improved Grapple (which is pretty much requisite to trying to grapple anyone at all)

2. Ranged weapons are easily negated, and Abundant Step is not that useful in this case
True, but few monks make active use of ranged weapons. The monk does best in a situation where she too can take to the skies and bring the game back to stunning and harassment (incidentally, Stunning Fist is an excellent recourse against primary arcane casters). How precisely the Monk is granted the ability to fly is, of course, variable. (items, potions, allies, etc.)
I agree that Abundant Step could use some work. If one were to ask me, I would make using the Abundant Step a move action.

3.Monks don't have the damage to take down a spellcaster and do little else to affect combat.
Monks are at a disadvantage for sheer damage output vs. fighters and other primary melee classes, true. I would contest that they do little else to affect combat; The Monk has mobility and resiliance on her side, making her difficult to kill by magical means. While she may not be able to fell the spellcaster with a few swift blows, she can provide her allies with valuable opportunities to act free of harassment. Every round the spellcaster spends trying to evade or shut down the monk is a round in which her teammates can contrive some means of dealing the telling blow or hampering the spellcaster enough to make the difference.

Clementx
2007-03-17, 12:48 PM
Pretty much all my melee characters have a locked gauntlet if their primary weapon can be disarmed. You have no significant advantage over the cleric for grapple checks and absolutely none over the druid.

Ranged weapons don't matter since they can be completely negated by a Wind Wall. Abundant step gets you to the caster and then waiting until next turn to act.

Yep, a cleric who needs a free hand to cast almost all his spells just LOVES having to take a full-round action to remove his mace from his locked gauntlet. The choice is either +10 to disarm, or a light shield with some juggling. Remove the shield AC to negate the disarming monk, and suddenly their AC drops by just the amount that monks are behind fighters in BAB, and he can pummel you better.

As for Wind Wall...what? Your party sorcerer is twiddling his thumbs? If your party has ranged-focused members, or melee-focused in case of a flying wizard, his first action should be to dispel magic it.

People always forget- if a magic spell is hard to negate physically, it can still be negated magically. If your opposing wizards are Batman, your party wizard should be one, too, or you just have a crappy party, and will lose no matter what.

martyboy74
2007-03-17, 12:48 PM
One further note about flight. If the Monk is already in your fries you can certainly take a 5-foot step and cast fly, but you can't thereafter fly away. You have a move action left, but something people often forget is that you can't move in a round that you take a five foot step. If you could, the withdrawal action wouldn't exist.
True. However, you can drop the Wall of X, then cast Fly, or, for more efficiency, just take the AoO of the casting of the spell. It's still pretty easy. From there, withdraw into the air.

Sir Giacomo
2007-03-17, 12:52 PM
He's not that bad, but he's not as good as many people think. The fighter is better with his high BaB and lessened MAD.


Just jumping in here at this point, agreeing to Yuki_Akuma. The fighter is better at handling fights vs casters, and even there he is very, very limited. Check out the thread where we're discussing the issue of core rules fighter capabilities vs casters, CR 20 monsters like a balor, complete with builds and loads of rules clarifications:
http://www.giantitp.com/forums/showthread.php?t=36333

You can actually get a lot of stuff out there for the question regarding whether monks are underpowered/useful/can fight casters well.

Now, elaborating on the reasons the fighter is more capable than the monk to fight casters:
- they have ranged weapons, in fact with the composite longbow from the beginning one of the most powerful ranged attacks there is in the game. Flying casters will have to do wind walls, protection from missiles etc. to keep away from that. (monks may spend a feat, though, for the martial weapon proficiency, but are behind in BAB and number of attacks, plus the archery specialisation needs a lot of feats).
- if (and that is a big IF) the opponents from level 10 or so do not have freedom of movement effects, the fighter is still more capable to grapple an opponent due to his higher BAB. In addition, of monk AND fighter have the same WIS score, a stunning fist attack by the fighter will be the same, plus the fighter again has a higher BAB and 1 more attack.
- the one thing the monk has in favour for him are the good will and reflex saves (with evasion), plus spell resistance. However, vs a SR at lvl +10 from lvl 13 onwards, casters may have already prayer beads (adding +4 to divine caster level) or ioun stones or other means to boost their caster levels, plus assay spell resistance spell if non-core (lowering SR by 10 without save), or spell penetration feats. In other words: the SR for the monk comes at higher levels, and has little use.
- there is an item out there called "monk's belt" which for a meagre 11,000 gp can give the AC bonuses of a monk (plus the ability to do real damage 1d8 with an unarmed strike) to all who wear it. So the fighter now has almost the same combat abilities as a monk. If the fighter takes the two-weapon fighting tree, he can even outflurry the monk (since it is done based on higher BAB).

All said, it appears as if the monk is the weakest class in core; which also reflects my game experience.

- Giacomo

Clementx
2007-03-17, 12:56 PM
True. However, you can drop the Wall of X, then cast Fly, or, for more efficiency, just take the AoO of the casting of the spell. It's still pretty easy. From there, withdraw into the air.
And then the monk uses his insane speed (and therefore Jump) modifier and saves to run around/over/through your obstacle, getting 20ft of movement in a row with no sweat, then grapple you in the air at the peak of his leap.

As for the AoO, he can try to Stun- which kills the wizard when the rest of the party catches up in the next round. So the wizard wouldn't risk it. So he has to step back and hover there for another monk Flurry. Or cast defensively, which is the smartest move, but leaves the monk close by (note the above mobility that is not just two-dimensional) and unimpeded.

Casters can expend all these spells to defend against the monk. And guess what? The monk has achieved his purpose, and in the next fight, he can do it all over again. Instead of Fireballing or Cloudkilling the rest of the vulnerable party, the caster has lost the initiative. And when casters have all these buffs up before the battle, it is harder for EVERYONE, not just the monk, hence the EL increase.

The Great Skenardo
2007-03-17, 01:02 PM
A fighter is certainly good at grappling, if he chooses to be, but he lacks (as Sir Giacomo notes) the Reflex or Will saves or even the touch AC of the Monk. I think this point is particularly important. As I see it, the monk is not meant to be able to out-damage a fighter. All the monastic training in the world won't enable you to do more damage in toe-to-toe combat than a trained fighter.
In my view, that isn't the point of a monk. In some senses, she is a more martial version of the bard, with a greater leaning towards rogue.
Depending on how you allocate skill points, a monk is often the best acrobat and a suberb stealth agent, as well as being good in a fight and extremely hardy against anything other than swords and clubs and claws. Her versiatility is her strong suit, because in almost any situation, there is something she can contribute.

EDIT: Clementx gets a cookie :smallsmile:

Tola
2007-03-17, 01:31 PM
Can you make a DC 400 jump check?

400?

Wait a minute...

*Looks at rules*

Says here 4xdistance is DC for a High Jump, right? Which would be the usual for going after something high up.

Basing this off the spell Fly.

Says here 'Flies at a speed of 60 feet, with good maneuverability', and 'Ascends at half that'.

So let's say our flying wizard takes his full movement(assuming that's possible, here) to get into the air, straight up.

Our monk(Or anyone else) wants to jump after him, and get him down to earth.

30x4=120.

....That's a hell of a Jump check. And that's WITH a running start. Without, it's 240.

Even if you account for the 8-foot reach of a Medium creature, tha's still 22x4=88. 176 from a standing start.

...Wow. And this at 5th level, possibly 3rd with this 'Alter Self'? Oy.

Bouldering Jove
2007-03-17, 01:32 PM
A wizard can be ready for any grapples just by having Dimension Door prepared.

Yuki Akuma
2007-03-17, 01:37 PM
Can you make a DC 400 jump check?

400?

Wait a minute...

*Looks at rules*

Says here 4xdistance is DC for a High Jump, right? Which would be the usual for going after something high up.

Basing this off the spell Fly.

Says here 'Flies at a speed of 60 feet, with good maneuverability', and 'Ascends at half that'.

So let's say our flying wizard takes his full movement(assuming that's possible, here) to get into the air, straight up.

Our monk(Or anyone else) wants to jump after him, and get him down to earth.

30x4=120.

....That's a hell of a Jump check. And that's WITH a running start. Without, it's 240.

Even if you account for the 8-foot reach of a Medium creature, tha's still 22x4=88. 176 from a standing start.

...Wow. And this at 5th level, possibly 3rd with this 'Alter Self'? Oy.

I chose 400 because 100 feet is a nice round number and still within range of most of the wizard's spells.

Weasel of Doom
2007-03-17, 04:58 PM
What would people say to skirmish of some sort instead of flurry and full BAB or keep flurry and replace fast movement with additional 5 foot steps

Clementx
2007-03-17, 06:01 PM
Last time I checked, this game was DUNGEONS and Dragons, not Wide Open Plains and Dragons. There are very few places in most dungeons for flight, especially where there is no where to climb/jump up towards that 30ft ceiling. For every time you create an environment biased for a wizard, you need to create one for the monk. Say, a 15ft high corridor lined with narrow beams over water. Let's see that wizard survive without Fly, and then take tactical advantage beyond, "Gee, I'm not drowning yet".

Bouldering Jove
2007-03-17, 06:17 PM
Last time I checked, this game was DUNGEONS and Dragons, not Wide Open Plains and Dragons. There are very few places in most dungeons for flight, especially where there is no where to climb/jump up towards that 30ft ceiling. For every time you create an environment biased for a wizard, you need to create one for the monk. Say, a 15ft high corridor lined with narrow beams over water. Let's see that wizard survive without Fly, and then take tactical advantage beyond, "Gee, I'm not drowning yet".
If you stage the battle in confined spaces, you're giving the wizard the ability to use spells like Wall of Force to completely prevent the monk from being able to reach him at all. The only environment I can imagine that's actually beneficial to the monk is if both combatants are in the same forcecage, and either within an Antimagic Field or with Dimensional Anchor already applied to both of them.

("Sunday, Sunday, Sunday! Two fantasy characters enter THE FORCECAGE, but only one man leaves! Metal! Magic! Mayhem! You paid for the whole seat, but you'll only need the EDGE!!")

MeklorIlavator
2007-03-17, 06:20 PM
Last time I checked, this game was DUNGEONS and Dragons, not Wide Open Plains and Dragons. There are very few places in most dungeons for flight, especially where there is no where to climb/jump up towards that 30ft ceiling. For every time you create an environment biased for a wizard, you need to create one for the monk. Say, a 15ft high corridor lined with narrow beams over water. Let's see that wizard survive without Fly, and then take tactical advantage beyond, "Gee, I'm not drowning yet".

How is Fly negated in that scenario? Also, isn't levitate a posiblity?
And why are you only playing in a dungeon. By your example, almost every fight should be vs. a dragon. In my campagns most battles take place outside because it gives everyone more options.

Jerthanis
2007-03-17, 06:27 PM
I've always felt like Stunning Fist was more of their signature ability than Flurry of Blows, and it coupled with Quivering Palm always made me think Monks were designed to be a sort of "save or suck" melee type that never really got off the ground. (both literally and figuratively)

I think if you want to increase their combat effectiveness, make some feats that allow use of stunning fists to do things like sicken, or fatigue enemies, perhaps make a Prestige Class that allows them to bestow negative levels with stunning fists, or drain stat points.

For ranged capability, they could get some sort of Chi-kung to strike from a distance or perhaps an ability to use Air Walk for a number of rounds a few times per day, allowing them to catch up with fliers if needed.

I've also thought they could really use Uncanny Dodge from a flavor standpoint. It feels weird that Barbarians can react to unseen threats easier than the Monk, who's entire concept is fighting using intuition.

Also, I really like using Tome of Battle classes, but every time I read them I have an overpowering urge to play Exalted.

TheOtherMC
2007-03-17, 06:33 PM
You know, there are some possibilities for fixing flurry. PH2 lets you swap it out for "Decisive Strike" which lets you full round for one attack at double damage. I cant say that'll be too usefull in the long run but at low levels, damn thats not bad. Plus theres also a feat in PH2 that lets monks shoot ki blasts...*Kamehameha*
:smallbiggrin:

Orzel
2007-03-17, 06:39 PM
The problem with monks is that it is outclassed by all the other roguish clasees in many situations.

The rogue has better and more skills. Rogues also tend to deal more damage since they don't need to full attack and hit with 50% of their attacks to deal good damage. They also get to use magic devices and open locks.

The ranger has more skills. Ranger's fulll BAB let them deal good consistent damage. They also get a pet, spells, free feat that don't stink after 4 levels.

The bard has more skills. Bards have spells to boost their attack damage or can deal spell damage. Did I mention they get spells? Yeah they get spells.

The problems of the monk is that:
1) It has 4 part MAD isssues (Str, Dex, Wiz, Con) and wants to have a 5th (Int) to fill party role.

2) It's damage relies on hitting with full attacks and not missing. But Monks are poor at staying alive at close range vs noncasters.

3) Although tough to stop with spells, it is not that hard to stop a monk with spells.

4) Monks aren't good at ranged combat compared to anyone who pump's their Dexterity score.

5) Many monk class features are silly or not very strong.

If I were to fix the class I'd:
make FoB just add bonus damage to attacks when moving, making AoOs, or full attacking.
Free TWF
Abundant step gives fly or DD once every 4 rounds

MeklorIlavator
2007-03-17, 06:57 PM
The problems of the monk is that:
1) It has 4 part MAD isssues (Str, Dex, Wiz, Con) and wants to have a 5th (Int) to fill party role.

2) It's damage relies on hitting with full attacks and not missing. But Monks are poor at staying alive at close range vs noncasters.

And they have 3/4 BAB, so they do miss more than a full melee class should. And I agree with you. Lord Iames was making a fix, but then he stopped updating.



3) Although tough to stop with spells, it is not that hard to stop a monk with spells.
That seems contradictory. Ithink you mean that they are hard to kill with spells, but not hard to stop with them.


4) Monks aren't good at ranged combat compared to anyone who pump's their Dexterity score.
Especially since they often do pump their Dexterity scores.


5) Many monk class features are silly or not very strong.

If I were to fix the class I'd:
make FoB just add bonus damage to attacks when moving, making AoOs, or full attacking.
Free TWF
Abundant step gives fly or DD once every 4 rounds
This is all very true.
I would suggest that the fix be done similar to Fax's Paladin rebuild in the idea of a modular class. That way Some of the Monk ablities that are contradictory could be found in thier own style. For example, a style that focuses on lots of smaller blows(skirmish+Flurry), or one that focuses on a divisive blow(Divisive Blow+Power attack), and one that focuses on Ki ablities for the DBZ fan (Ki blast+constipation) .

TheOtherMC
2007-03-17, 07:02 PM
Other nifty PH2 feats include igniting your fists in ki flame for extra fire damage and for those hardcore fluffers out there, "Versitile Strike" which allows you as a swift action to change the type of damage your unarmed strikes deal. (ie. from slashing->piercing->bludgeoning->repeat)

Now here's the kicker. Imagine Versitile strike with enchanted vorpal fists, possibly Kensai. YOU COULD CHOP SOMEBODY'S HEAD OFF WITH YOUR BARE HANDS!

/end rant

MeklorIlavator
2007-03-17, 07:05 PM
Other nifty PH2 feats include igniting your fists in ki flame for extra fire damage and for those hardcore fluffers out there, "Versitile Strike" which allows you as a swift action to change the type of damage your unarmed strikes deal. (ie. from slashing->piercing->bludgeoning->repeat)

Now here's the kicker. Imagine Versitile strike with enchanted vorpal fists, possibly Kensai. YOU COULD CHOP SOMEBODY'S HEAD OFF WITH YOUR BARE HANDS!

/end rant
I would rather go with throwing+collision+flaming.
Rocket punch rules. Plus, you then go and pick up your hands and reattach them. Whats not to like?:smallbiggrin:

NemoUtopia
2007-03-17, 07:10 PM
Let's see, Monk has at least 40ft. speed by the point the Wiz is flying, and if the Wiz can fly like that, then you can bet your bippy the monk can get a running start. So DC effectively becomes distancex2 - 8ish. The party wizard has cast Jump or the monk has some access to this boon by magic item (easily affordable potion). So we're looking at effectively having 8 ranks + 30 magic + (we'll be nice and say 2 for Str) 2 stat = 40 x 4/3 = 53.333..., + 8ish = ~60 ft. +d20 die roll = ~70ft. A monk can catch a Wizard who has ascended using Fly in the given scenario, and sometimes doesn't even need the easily accessible boost. The boost becomes moot at higher level as well, since the monk's speed will have increased. That said, monks seem to be fun (and are fun, at that), but find themselves in the bard's position of "I'm-not-a-rouge-or-a-fighter-or-a-sorcerer-much-less-a-wizard-or-cleric-etc." I like playing them, but mostly from the look on my DM's face when I make pretty much every save and the enemy casters can't hit my touch AC.

DM: Alright, now save vs...
Monk: Is this high enough?
DM: :smallfurious: dammit!
Monk: :smallcool:
DM: Fine the wizard hits you with ray of...
Monk: Did he hit my TOUCH AC of 20+?
DM: :smallfurious: dammit!
Monk: :smallcool:

martyboy74
2007-03-17, 07:18 PM
DM: Alright, now save vs...
Monk: Is this high enough?
DM: :smallfurious: dammit!
Monk: :smallcool:
DM: Fine the wizard hits you with ray of...
Monk: Did he hit my TOUCH AC of 20+?
DM: :smallfurious: dammit!
Monk: :smallcool:
DM: The wizard disintegrates the ground beneath you. Roll Xd6 points of damage.
Monk::smallfurious: dammit!
DM: Oh yeah, he also creates a solid fog in the pit....
Monk::smallfurious: dammit!
DM: And he creates a Wall of Iron above the pit. Jump out of that, monkey boy.:smallyuk:

TheOtherMC
2007-03-17, 07:18 PM
I would rather go with throwing+collision+flaming.
Rocket punch rules. Plus, you then go and pick up your hands and reattach them. Whats not to like?:smallbiggrin:

Well THEN you throw your ki blast FROM your now stumpy arms *Hell Flash!*
:smallbiggrin:

None of these abilities say you need fists ATTACHED at the time, just that you give up one or two uses of stunning fist (Though to be fair taking these feats gives you that many more uses per day anyway.

MeklorIlavator
2007-03-17, 07:27 PM
You know, there is something better with the kensia. See, the monk saya that you can use any part of your body for unarmed strike. The Kensia allows you to enchant an attack of yours.

So enchant your head with throwing+whatever. So now your head goes and attacks the person and your headless body is throwing Ki blasts around.


Or enchant it with throwing and flaming, and make it so your head looks like a pumpkin. Yes, you are the headless horseman. Gotta do that one of these days.

NemoUtopia
2007-03-17, 07:27 PM
DM: The wizard disintegrates the ground beneath you. Roll Xd6 points of damage.
Monk: good thing I have slow fall :smallcool:
DM: Oh yeah, he also creates a solid fog in the pit....
Monk: That's fine, I've already used my action to jump out of the pit and grapple him
DM: :smallfurious: dammit!

Corrected for your viewing pleasure :smallwink:


Edit: Oh yeah, and for the DBZ-esque thing, you probably want to go Enlightened Fist. It's less HP, but more "Ki", and lots of fun!

Khantalas
2007-03-17, 07:30 PM
Corrected for your viewing pleasure :smallwink:

Oh, think about it. Slow fall, where there is nothing to slow your fall?

You're in the middle of a pit, with the closest wall being about 10 ft. away.

You also have to run to make a meaningful jump.

martyboy74
2007-03-17, 07:35 PM
DM: The wizard disintegrates the ground beneath you. Roll Xd6 points of damage.
Monk: good thing I have slow fall :smallcool:
DM:Doesn't that only work if there's a wall with in reach?
Monk:...
DM: Oh yeah, he also creates a solid fog in the pit....
Monk: That's fine, I've already used my action to jump out of the pit and grapple him
DM: Sorry, he used Celerity to go before you, and the solid fog exists.
Monk::smallfurious: dammit!

Corrected for an intelligent wizard, and the rules.

Orzel
2007-03-17, 07:37 PM
That seems contradictory. Ithink you mean that they are hard to kill with spells, but not hard to stop with them.


Yeah that's what I meant. They eat death and damage spells for breakfast but trap them via a nontargeting spell and the fight gets easy.

Monks don't fill any of the typical non-skill party roles well. They are just there and don't die fast.

MeklorIlavator
2007-03-17, 07:51 PM
Monks don't fill any of the typical non-skill party roles well. They are just there and don't die fast.
Actually, they do comic relief quite well(see my most recent monk/kensia posts)

cupkeyk
2007-03-17, 08:10 PM
By the time the monk can afford a +1 shocking shuriken the wizard will have 13 ranks n concentration. Wheee.

And no, the monk is generally not better than any other melee combatants againts a wizard. The best melee threat to a wizard is a full attacking halfling outrider that will win the initiative gets a surprise round and then flying charging full attacking and killing the wizard before the wizard gets his first action. The best noncaster against a wizard is actually a archer fighter who can multi-shot the wizard to death ON the surprise round.

Monks are good at running away.

And if the wizard can't win the day, that wizard was built wrong and is played by a stupid person.

greenknight
2007-03-17, 08:26 PM
I've heard it around here that there are problems with this class.

The title really says it all.

(Actually, while we're at it, we should say what's 'wrong' with other classes as well.)

To me, the Monk is meant to be a Mage (Wizard or Sorcerer) killer, and they do that fairly well if they can get to the Mage and past the Mage's defences. The problem is, they have trouble doing that. In order to balance the Monk, here's what I suggest:

Start by removing some silly or unbalancing abilities:

1) Drop the Monk's Unarmored Speed Bonus so that it improves only 5' each time rather than 10'. This is a balance issue to prevent a situation where a Monk can just Spring Attack/range attack other physical attackers without them being able to counterattack.

2) Give Tongue of the Sun and Moon to Druids. There's no real reason why it should be a Monk ability, and it makes much more sense for a Druid to have it.

3) Get rid of Purity of Body, Wholeness of Body, Diamond Body and Perfect Self. The amount of benefit these give is fairly small, and we're going to be giving the Monk a lot of other high powered goodies so something has to go for balance.

Now power up with:

1) Flurry of Blows has no penalty to attack from level 5. (EDIT: Changed from Level 1 to prevent some potential cheese from someone taking 1 level of Monk)

2) Instead of Purity of Body, give the Monk Air Walk (as the spell, but 1 rnd/lvl and self only), 1x/day as a free action. This increases to 2x/day at level 10, 3x per day at level 15 and 4x/day at level 20. This gives a Monk some ability to deal with flyers.

3) Instead of Wholeness of Body, give them the ability to Enlarge Person (as the spell, but 1 rnd/lvl, self only, and works on any creature type), 1x/day, increase to 2x/day at 12th level, 3x/day at 16th, and 4x/day at 20th. At 15th level, the Monk can choose to use two uses to get a Greater Enlarge Person effect (1 rnd/lvl, self only, works with any creature type, increase 2 size categories, +8 size modifier to Str, -4 size modifier to Dex, +4 size modifier to Con, -2 size modifier to AC and attacks). This gives a Medium sized Monk Reach, and a bit more damage per round. This isn't really that great a boost, because it comes with a loss of Dex, AC and accuracy. But it is necessary to defeat the 5' step tactic, and it helps a lot with Disarm and Sunder attempts.

4) Give them Combat Reflexes or Improved Sunder as a bonus Feat at level 8. This allows the Monk to be an AoO monster, or overcome some of the problems of a Locked Gauntlet.

4a) EDIT: Give a Dimensional Anchor type ability at 9th level. This must be made as a melee attack (not an action, cannot be combined with Stunning Fist), 1x/day, caster level equal to Monk level. Increase to 2x/day at 12th, 3x/day at 15th, 4x/day at 18th.

5) At 10th level, give them the ability to use Greater Dispel Magic 1x/day (using their Monk level as Caster level) as a standard action. Increase to 2x/day at 14th level, 3x/day at 18th level. At 15th level, make it a Free action. This should help them remove at least some spellbuffs and magical obstacles.

6) Replace Diamond Body with an Elemental Damage effect (+1d6 damage of acid, cold, electricity, fire, or sonic - Monk's choice). Activation is a free action and lasts 1 round, but replaces 1 stunning attack per use (and requires that the Monk have Stunning Fist). At the cost of 2 stunning attacks, the Monk can add 1d10 damage on a Critical Hit.

6a) Edit: At 10th level, give the Monk a constant Nondetection ability (DC 15+ Monk level). This is in line with the Stealth aspect of the class.

7) Have Abundant Step increase uses with level - 2x/day at 16th, 3x/day at 20th.

8) Allow Quivering Palm to be used 1/day.

9) At 17th level, replace Tongue of the Sun and Moon with Mage's Disjunction (1/day, standard action, save DC based on Wisdom), caster level equal to Monk level. This automatically removes all spellbuffs, and probably wipes out a number of magical items as well. Mages, and foes who rely heavily on magical items but have a low Will save, will FEAR a Monk with this ability.

Logos7
2007-03-17, 08:40 PM
Yeah i think Thousand Faces and Tounge of the Stars and Moon need to be switched. Never saw druids as the incognito bits for the fact they could look like whoever they want so much as for the fact they are all aloof and mysterious and crap

I would like to see Wuxia as abosed to the absordish mishmash, drop some of the weirder abilities, and get more uses of Aboundant step as a free action and make quivirering palm a capstoner with the Outsider Type as aposed to a crappy once every now and then ability

Logos

Arbitrarity
2007-03-17, 08:58 PM
Let's see, Monk has at least 40ft. speed by the point the Wiz is flying, and if the Wiz can fly like that, then you can bet your bippy the monk can get a running start. So DC effectively becomes distancex2 - 8ish.

Whaa?

It's distance x4 base, x8 from standstill.

8ish my rear. Monk with his fearsome 40 ft speed, and 5 ft step run, vs wizard flying 60 ft up.

"I jump"
"Jump check please!"
"Um.... (Assumed idiotic, minmaxing jump build) 46(ish) plus natural 20! 66!"
"Did you beat DC 240?"
"No..."
"You fail. The wizard laughs, and remembers he is level 3. He stops laughing, and magic missiles you. You take 21 damage, as you can't reach him You die. Why he prepared 3 magic missiles I don't know, but why you're a thri-kreen monk who is attempting to jump 60 feet up I don't know."

NemoUtopia
2007-03-17, 08:58 PM
Corrected for an intelligent wizard, and the rules.

:smallsigh: oh well, I guess I'll have to point out things I neglected before because it would annoy the Wizard players:

1) Monk is entitled to reflex save and jump check after "floor falls out" (if you don't believe me, you're mis-reading the rules, trust me)
2) Regardless, monks can Climb
3) Not to mention (at that level) Abundant Step
4) and all of that is assuming said monk is sans magical item X of flying/levitation [and is not already using it and just happens to be above the disintegrated floor]

I can list a thousand other things, but the end point is (and this is coming from a Wizard player): Short of saveless/SRless spells, Monks have an advantage, unless the wizard has every save/SR booster available to the exclusion of other feats [read: specialized case]. That is, until the wizard lands a successful forcecage :smallbiggrin:

greenknight
2007-03-17, 09:26 PM
Corrected for an intelligent wizard, and the rules.

You do know the rules for Disintegrate vs an Object, right?

From the SRD:

When used against an object, the ray simply disintegrates as much as one 10-foot cube of nonliving matter.

Since the Monk was standing in that area, there will be a wall within arm's reach, and Slow Fall will work. Even if there isn't a wall, it's only a 10' drop, and Tumble should allow the Monk to ignore that.

Caelestion
2007-03-17, 09:33 PM
Particularly as these wizards always seem to be casting fifty spells at once and at any given second. I know they're powerful (Celerity-type spells should be exclusively chronomancy), but still.

AtomicKitKat
2007-03-17, 10:04 PM
2) Give Tongue of the Sun and Moon to Druids. There's no real reason why it should be a Monk ability, and it makes much more sense for a Druid to have it.

Beats me. It's mostly so they don't have to spend their already pitiful stock of skill points(pitiful only because they want things like Tumble, Hide/Move Silently, Listen/Search/Spot, and MAD lowers their Int) on Speak Language I guess.


3) Get rid of Purity of Body, Wholeness of Body, Diamond Body and Perfect Self. The amount of benefit these give is fairly small, and we're going to be giving the Monk a lot of other high powered goodies so something has to go for balance.

Perfect Self is crap(but see below). The rest at least let him stand toe to toe with some nasties(pretty much anything that gives disease or poison on their attacks. Unless it's supernatural crap. Then he's screwed.)


Now power up with:

1) Flurry of Blows has no penalty to attack from level 5. (EDIT: Changed from Level 1 to prevent some potential cheese from someone taking 1 level of Monk)

Change it to a style thing. Perhaps the higher speed monks(with better than 5 foot speed increases) get it. Or make it so they get to do 2 swings(at Full AB with -1 to -2 penalty) on any attack action.


2) Instead of Purity of Body, give the Monk Air Walk (as the spell, but 1 rnd/lvl and self only), 1x/day as a free action. This increases to 2x/day at level 10, 3x per day at level 15 and 4x/day at level 20. This gives a Monk some ability to deal with flyers.

This feels more like a replacement for Abundant Step.


3) Instead of Wholeness of Body, give them the ability to Enlarge Person (as the spell, but 1 rnd/lvl, self only, and works on any creature type), 1x/day, increase to 2x/day at 12th level, 3x/day at 16th, and 4x/day at 20th. At 15th level, the Monk can choose to use two uses to get a Greater Enlarge Person effect (1 rnd/lvl, self only, works with any creature type, increase 2 size categories, +8 size modifier to Str, -4 size modifier to Dex, +4 size modifier to Con, -2 size modifier to AC and attacks). This gives a Medium sized Monk Reach, and a bit more damage per round. This isn't really that great a boost, because it comes with a loss of Dex, AC and accuracy. But it is necessary to defeat the 5' step tactic, and it helps a lot with Disarm and Sunder attempts.

Instead of enlarging themselves(which really ruins the "flavour"), make it so they can increase their reach as an immediate action. This allows them to "pre-empt" the Wizard's 5 foot step, and smack him upside of the head before he can cast.


4) Give them Combat Reflexes or Improved Sunder as a bonus Feat at level 8. This allows the Monk to be an AoO monster, or overcome some of the problems of a Locked Gauntlet.

There's an Eagle Claw feat somewhere that gives bonuses to sundering. Probably give them that. Agree that they need a couple of bonus feats more somewhere.


4a) EDIT: Give a Dimensional Anchor type ability at 9th level. This must be made as a melee attack (not an action, cannot be combined with Stunning Fist), 1x/day, caster level equal to Monk level. Increase to 2x/day at 12th, 3x/day at 15th, 4x/day at 18th.

Maybe. Alternatively, as part of their grapple, allow them to expend a use of stunning fist for this.


5) At 10th level, give them the ability to use Greater Dispel Magic 1x/day (using their Monk level as Caster level) as a standard action. Increase to 2x/day at 14th level, 3x/day at 18th level. At 15th level, make it a Free action. This should help them remove at least some spellbuffs and magical obstacles.

As above. Except probably as part of an attack. "I hit you so hard, the magic flies off you!"


6) Replace Diamond Body with an Elemental Damage effect (+1d6 damage of acid, cold, electricity, fire, or sonic - Monk's choice). Activation is a free action and lasts 1 round, but replaces 1 stunning attack per use (and requires that the Monk have Stunning Fist). At the cost of 2 stunning attacks, the Monk can add 1d10 damage on a Critical Hit.

Elemental with burst? Flavourful. Could probably do with adding it to a style. Maybe each elemental school/style gets 2 powers. One for a hit/burst, one for a one round per 2 levels Fire Shield effect(with corresponding damage). Maybe not Sonic. Unless you feel like using the Lion's Roar. :smallwink:


6a) Edit: At 10th level, give the Monk a constant Nondetection ability (DC 15+ Monk level). This is in line with the Stealth aspect of the class.

Seems fair enough.


7) Have Abundant Step increase uses with level - 2x/day at 16th, 3x/day at 20th.

Doesn't change the fact that they still can't do a damn thing after this ability. I vote dropping it for Air Walk.


8) Allow Quivering Palm to be used 1/day.

As written, it's more of an ability the DM can use to say "I don't like your character. Shin de kudasai(Please die)." Replacing it with a nauseating punch or similar(see Complete Warrior's various feats like Freezing the Lifeblood, Weakening Blow, etc.) makes it more useful in combat.


9) At 17th level, replace Tongue of the Sun and Moon with Mage's Disjunction (1/day, standard action, save DC based on Wisdom), caster level equal to Monk level. This automatically removes all spellbuffs, and probably wipes out a number of magical items as well. Mages, and foes who rely heavily on magical items but have a low Will save, will FEAR a Monk with this ability.

Seems alright to me. Your party will hate you though. :smalltongue:

I'd adjust Perfect Body so that you get DR 15/Magic and Lawful and Adamantine. This way, only a fellow Monk of the same level would be able to hit you consistently. On that thought, adjust it so that they get the DR at the same time as the Ki Strikes.

Ki Strike(Magic)+DR 5/Magic
Ki Strike(Lawful)+DR 10/Magic and Lawful
Ki Strike(Adamantine)+DR 15/Adamantine, Lawful and Magic.

I am unsure whether it should be increased at level 20(DR 20/A+L+M seems a little strong, but oh so appropriate. :smallbiggrin:)

While we're on the topic of funny builds, here's one from ToB:

http://boards1.wizards.com/showthread.php?t=757496

greenknight
2007-03-17, 11:03 PM
Beats me. It's mostly so they don't have to spend their already pitiful stock of skill points(pitiful only because they want things like Tumble, Hide/Move Silently, Listen/Search/Spot, and MAD lowers their Int) on Speak Language I guess.

1st Ed AD&D Monks had something along these lines (restricted to animals and plants), so I guess it's traditional. But still, it doesn't really mesh with the Monk's other abilities.


Perfect Self is crap(but see below). The rest at least let him stand toe to toe with some nasties(pretty much anything that gives disease or poison on their attacks. Unless it's supernatural crap. Then he's screwed.)

Exactly. Sometimes beneficial, but I thought it would be best to clear the decks for other stuff which allows the Monk to work better.


Change it to a style thing. Perhaps the higher speed monks(with better than 5 foot speed increases) get it. Or make it so they get to do 2 swings(at Full AB with -1 to -2 penalty) on any attack action.

I've seen high level Spring Attacking Monks using RAW, and they are virtually impossible for physical attackers to counter due to their high speed, Deflect Arrows and the ability to Sunder or Disarm a ranged weapon. Slowing them down a bit gives the physical attackers a chance, and doesn't hurt the Monk's core capabilities much.


This feels more like a replacement for Abundant Step.

Difference is, Abundant Step allows the Monk to go through barriers and ignore Wind effects. What I needed here was something which allows a relatively low level Monk to melee with flyers, and Air Walk seemed to suit the Monk's style quite well.


Instead of enlarging themselves(which really ruins the "flavour"), make it so they can increase their reach as an immediate action. This allows them to "pre-empt" the Wizard's 5 foot step, and smack him upside of the head before he can cast.

Maybe. There's lots of stories of characters who enlarge themselves when fighting a foe, and it just seems better to me than have the Monk have rubber arms. That said, I can see an alternative path Monk giving up Enlargement to be able to do this and get a bonus to Escape Artist checks.


There's an Eagle Claw feat somewhere that gives bonuses to sundering. Probably give them that. Agree that they need a couple of bonus feats more somewhere.

Eagle Claw Attack (from Complete Warrior, and Sword & Fist I think) gives a Wisdom bonus to damage on a Sunder, but it doesn't avoid the AoO or bonus to attack that Improved Sunder gives, so I'd rather go with that as a bonus. The Monk can then select Eagle Claw Attack as a regular Feat, since Power Attack isn't a specific pre-req for it (it is needed for Improved Sunder, unless the Monk gets it as a bonus).


Maybe. Alternatively, as part of their grapple, allow them to expend a use of stunning fist for this.

Problem is, some magic prevents a grapple from working (Freedom of Movement). I thought about tying this with Stunning Fist, and it could be done that way, but I wanted much fewer uses per day without burning 3 or 4 Stunning Fist uses.


As above. Except probably as part of an attack. "I hit you so hard, the magic flies off you!"

The reason I don't do this is because there are so many different types of magical effect a Mage can produce, some of which are just obstacles to movement (eg, the oft quoted Solid Fog). I want the Monk to have a chance at getting rid of them (as a free action, at higher levels) and then be able to move into melee. I was considering giving Monks Freedom of Movement, but I think the Monk should get a magical item for that, since it's such a great ability for them.


Elemental with burst? Flavourful. Could probably do with adding it to a style. Maybe each elemental school/style gets 2 powers. One for a hit/burst, one for a one round per 2 levels Fire Shield effect(with corresponding damage). Maybe not Sonic. Unless you feel like using the Lion's Roar. :smallwink:

Different styles are a pretty good idea. Sonic's ok, because it could be explained as a vibrating attack creating shockwaves.


Doesn't change the fact that they still can't do a damn thing after this ability. I vote dropping it for Air Walk.

This would be more as a guaranteed way to get past obstacles quickly. Which is why it's a higher level effect which is available less often.


As written, it's more of an ability the DM can use to say "I don't like your character. Shin de kudasai(Please die)." Replacing it with a nauseating punch or similar(see Complete Warrior's various feats like Freezing the Lifeblood, Weakening Blow, etc.) makes it more useful in combat.

It's supposed to be the Monk's most feared ability (although IMO I've gone one better with Disjunction), so Save or Die seems appropriate, especially since the casters have so many Save or Die spells per day already by this level.


I'd adjust Perfect Body so that you get DR 15/Magic and Lawful and Adamantine. This way, only a fellow Monk of the same level would be able to hit you consistently. On that thought, adjust it so that they get the DR at the same time as the Ki Strikes.

I was thinking of giving the Monk something like Righteous Might here, with DR 10/Chaotic (Chaotic because creatures of opposite alignment seem to be the ones which bypass DR - see Righteous Might and the DR of Devils, Demons and Angels for example). I decided against it because so many other abilities I've described give an additional use at level 20, so I thought it would be a bit over the top. Either that, or give more uses of Enlarge Person at higher levels and make the double cost ability similar to Righteous Might. Hmm, come to think of it, that's not a bad idea at all....

Bears With Lasers
2007-03-17, 11:14 PM
You know, that's completely freakin' counterintuitive. Shouldn't you be getting DR that protects you from your sworn enemies, rather than from everyone who isn't your enemy?

greenknight
2007-03-17, 11:19 PM
You know, that's completely freakin' counterintuitive. Shouldn't you be getting DR that protects you from your sworn enemies, rather than from everyone who isn't your enemy?

I know that, but that's how it seems to be in the game. Couldn't agree more though that it's a rather silly way to do it. But at least it's not DR X/magic, which practically everyone seems able to ignore at higher levels.

Draco Ignifer
2007-03-17, 11:21 PM
Turn it around - shouldn't your sworn enemies, who've spent their entire existances trying to fight your kind, have evolved or otherwise developed a natural mechanism to break through your defenses? Kinda like how everything's afraid of the poison certain animals put out except for one type of creature which preys exclusively on them and is immune.

Hence, why demons are dr /good - not because they don't want to be protected from good, but because good has learned how to pierce demonic resistances. And, hence, why chaotic creatures would know how to pierce the spiritual armor of a monk.

Bears With Lasers
2007-03-17, 11:23 PM
Yeah, I know how it is. I just never really thought about it before. "Congratulations, cleric! The protection from your spell, which your god wants you to use to fight demons, will... apply to everyone except demons! lawlz!"

As for monks, they're a little better off against spellcasters than other magicless melee types. But that's a raw deal, because while a lot of the enemies you fight in D&D have magic, SLAs, et cetera, very few of them are actual squishy spellcasters. A Black Wyrm is probably the toughest CR 20 monster as written; in Core, at least, if not with every splatbook. It's a 13th-level caster. Do you really think it's going to care about your AoO grapple attempt?

Monks are heavily melee-inclined--stunning fist, flurry of blows, Improved Trip, yadda yadda--but the problem is, they're crap at it. Level 10 monk vs. Hill Giant! Dead in a couple of rounds barring tons of buffs and AC-whoring that means your offensive ability is absolutely nil.

There are a bunch of ToB maneuvers that would make great per-encounter or even at-will abilities for the monk. Most of them are Setting Sun, like the throws; then there's the Setting Sun counter that lets you follow someone after they five-foot-step away as an immediate action, and the one that lets you follow someone who takes a *move action* to get away.

...man. Every time I consider fixing the Monk, I realize that the Swordsage is the monk as it ought to be. Swordsages freakin' rock.

greenknight
2007-03-17, 11:36 PM
[quote]Monks are heavily melee-inclined--stunning fist, flurry of blows, Improved Trip, yadda yadda--but the problem is, they're crap at it. Level 10 monk vs. Hill Giant! Dead in a couple of rounds barring tons of buffs and AC-whoring that means your offensive ability is absolutely nil.

You're right, and at higher levels even getting into melee with your foe is so hard I sometimes wonder why the Monk left the Monastery. That's why my suggested changes to the class allows them to enter melee more easily, and do a bit more damage when they get there. They'll still need magical items (an Amulet of Mighty Fists and a Ring of Freedom of Movement seem like natural choices), but at least they're a little better at what they are supposed to do.

Stephen_E
2007-03-17, 11:54 PM
What's this insane Monk speed people are talking about. The Barbarian is faster at low levels, and even when the Monk catchs up, he's still worse at jumping bacause he doesn't have the raging Barbarians strength.
And people conviently seem to forget that because the Monk has to tumble to get past the mages guards, he only has 1/2 movement which means he won't have a readied action after he's used a double move to get to the mage (assuming he didn't bounce off someone when trying to tumble through a enemy occupied square.

The Monk build I've seen that worked was a Monk with the Ebberon feat that allowed him to use a Long Spear as a Monk Weapon and got Power Attack, Stand Still, Elusive Target and Improved Disarm. The Awrchanter in the party would use Reckless Song, to allow him to reduce his AC to boost his att, upto his BAB. With Flurry+Boots of Speed, that was a real nasty Monk.

The build I think would work is the Monk/Master Thrower Shuriken combos, with 1 lev of Artificer to allow infusing 50 shuriken with Bane whatever you're fighting.

In general the problem with Monks IMHO is that the entire class is (as other have pointed out) MAD. They need 4-5 high stats, High BAB, High hit points, High AC. Barring incredible rolls, they don't have the stats, the class doesn't give them enough BAB, AC or Hit points. They also have a stupid alignment restriction (of course without it you'd see more 1st lev Barbarian/Monks, but I could live with that).

The strong point of Monks. They're funny. Lots of roleplaying potential.

Stephen

NemoUtopia
2007-03-18, 12:22 AM
......
The strong point of Monks. They're funny. Lots of roleplaying potential.

Stephen

yep, pretty much. But, as you said, the Eberron feats can do a bit to beef up monks, getting the martial weapon as a monk weapon and feats that apply to it. I had fun with a twin-sword monk once, but it was mostly since the rest of the party was "optimum-min-max" at the time, so the challenges weren't the kind to take out the super party.

Ramza00
2007-03-18, 01:17 AM
It seems well handled already but here is another 5 page thread with similar comments.
http://www.giantitp.com/forums/showthread.php?t=32086

Me listing like 30 monk prcs is on the third page. Sadly almost all of them suck or you are a monk/X hybrid character

Krellen
2007-03-18, 10:18 AM
What's this insane Monk speed people are talking about. The Barbarian is faster at low levels, and even when the Monk catchs up, he's still worse at jumping bacause he doesn't have the raging Barbarians strength.
No, he just has a +4 bonus for each 10' bonus to his speed. At 20th level, a Monk has a +24 bonus to Jump, just from his speed. That far outstrips the Barbarian's +8 from speed and raging strength. So at levels 1-5, the Barbarian is a better jumper, but after level 6, the Monk is the better jumper.

A 20th level monk can jump 5' straight up from a stand still (allowing him to grab at things 10' up) without even rolling the die (I assume the Monk puts ranks in Jump; it's practically required for the flavour of the class).

Hzurr
2007-03-18, 11:07 AM
I think everyone is argueing some pretty specific things here. Let's back up a bit.

1. Stats - More than any other character, a monk needs all good stats (excluding Cha) to do what a monk is designed to do. You might be able to get away with a low int.

2. To-hit Flurry of blows is great against the high HP, low AC creatures. Where monks run into trouble is when they find themselves in a situation where the thing they're fighting has high AC. Statistically, they can flurry of blows all day long, but they'd be better off with a high to-hit and half as many attacks.

3. Grapple (and trip, and disarm) - The reason people say monks are good at grappling, is because they can take the feat improved grapple. That's it, they don't have any other bonuses. Same goes for Tripping, or disarming. A fighter who took the feat "improved grapple" or "Improved Trip" is better at it than a monk is

4. Weapons For a monk to be able to use a weapon, he sacrifices all the spiffy unarmed-attack bonuses, which defeats the purpose. To enchant his natural weapons, a monk has to spend a fortune on an "amulet of mighty fists," and still caps out at a +5. (which costs almost as much as a +9 regular weapon), and never gets any special abilities, just the pure +5 (a monk never has the option of something like "Bane" or "Vorpal" or "Ghost Touch".) Without trying to do something cheap like "Greater Magic Fang -Permincany," this is a serious disadvantage.

5. Range - A monk's "ranged weapon" is "I have a 60 ft movement, and I'll just get there." That's about it. If that doesn't work, they have a sling (whoop-ti-do), and shuriken (with a whole 10 ft range, and 1d2 damage)


Now, having said all that, let me establish that I really like Monks. I enjoy playing them (assuming I've rolled decent stats), and they have some really fun abilities. Yeah, sometimes I feel ineffective, but I have a blast playing them regardless.


Also, why is everyone talking about monks being able to jump? The only "jumping ability" they have is that its a class skill. We aren't talking ninjgas here people.

As far as monks and casters, they're only advantage is that they have good saves, and hopefully they can reach the caster quickly and stunning fist him/her. If those things don't work...they're kind of screwed.

- Ah, Just read the thing about speed and jumping. Didn't actually know about that. Spiffy.

- Also, Amulet of Might Fists takes up the same slot as a periapt of wisdom would, so you can't do both

Rigeld2
2007-03-18, 12:20 PM
As far as monks jumping and grappling low level casters?

Go for it. Clerics of the Travel domain make you slip right off and fall to the ground. Wizards dont have as ready access to Freedom of Movement, but they can Dimension Door out pretty easily.

Tola
2007-03-18, 12:33 PM
5. Range - A monk's "ranged weapon" is "I have a 60 ft movement, and I'll just get there."

70-120 foot movement, to be fair. I can't think of many situations, barring large amounts of blockages, where he CAN'T get to where he wants to go. 14 squares at first to 24? 25? at max. The problem is he really can't use it to it's utmost. As has been said, he can't really handle opponents on his own.

Some sort of physical/magic deflecting(or perhaps even nullify+returning?) ability would NOT go amiss. If he's got this obscene speed to get into the heart of the battle quickly, then dammit, he should be able to hold his own till help arrives. There's nothing wrong with being the one to hold attention, but one wants to make a decent dent before you charge away again.

Let's see...

Deflect Blows.

Monk Level....7? 9?

Any melee strike against the monk allows him to make a special grapple check, using both your Strength and Wisdom modifiers. If you lose, the attack connects as normal. If you win, the attack is deflected entirely, stopping any iterative attacks dead. Win by 5 or more, and you may force the opponent into any clear square next to you. Win by 15 or more and you may take a free attack at your foe as you move him.

Horribly flawed, I imagine, but the thought might be clear enough.

daggaz
2007-03-18, 12:46 PM
Well, not exactly about the monk, but it was mentioned earlier and I cant believe nobody brought up this point...


About a cleric using a locked gauntlet to prevent monk disarmage... It doesn't screw up spellcasting or touch attacks in ANY way. You need one hand to cast spells, and one hand to make touch attacks. Thats it. So feel free to lock up a hand to your weapon. As a dwarven cleric, Im always using a twohander anyways, so might as well lock up my offhand to it, just in case the DM actually gets tedious about doing mundane things one handed in the middle of things..

AtomicKitKat
2007-03-18, 01:12 PM
Well, not exactly about the monk, but it was mentioned earlier and I cant believe nobody brought up this point...


About a cleric using a locked gauntlet to prevent monk disarmage... It doesn't screw up spellcasting or touch attacks in ANY way. You need one hand to cast spells, and one hand to make touch attacks. Thats it. So feel free to lock up a hand to your weapon. As a dwarven cleric, Im always using a twohander anyways, so might as well lock up my offhand to it, just in case the DM actually gets tedious about doing mundane things one handed in the middle of things..


Indirectly I think. Was mentioned that most Clerics are using their "one" free hand on a shield(well, any shield heavier than "light").

Morty
2007-03-18, 01:16 PM
Does anyone else have impression that whole idea of someone focusing on fist fighting is silly most of the time? You can beat humanoids to death, but I just can't imagine anyone killing dragons or demons by kicking and punching. That's why I support the idea of replacing monk with swordsage- it's the only case when replacing core class with ToB one.

MeklorIlavator
2007-03-18, 01:31 PM
Indirectly I think. Was mentioned that most Clerics are using their "one" free hand on a shield(well, any shield heavier than "light").
Well, shields technically don't lock up a hand, but they give arcane spell failure. So you could have a shield and still cast spells, as long as you are also holding a holy symbol, which could be part of a weapon or shield.

Maxwell
2007-03-18, 01:55 PM
Does anyone else have impression that whole idea of someone focusing on fist fighting is silly most of the time? You can beat humanoids to death, but I just can't imagine anyone killing dragons or demons by kicking and punching. That's why I support the idea of replacing monk with swordsage- it's the only case when replacing core class with ToB one.

It's not silly. Killing a dragon with a sword would be just as unrealistic. Imagine someone one tenth your size trying to kill you with a pin, thats what fighting a dragon would be like. This is just the way DnD works. However, I have never read the entry for the Swordsage, and for all I know it could be the perfect solution.

Clementx
2007-03-18, 02:26 PM
Well, shields technically don't lock up a hand, but they give arcane spell failure. So you could have a shield and still cast spells, as long as you are also holding a holy symbol, which could be part of a weapon or shield.

Read the description of light and heavy shield again. You can't complete somatic or material components with it, and having your holy symbol on them is not RAW. Holy symbols are divine focuses, and by spellcasting rules, you need a hand free for focuses (which can be the same for material and somatic).

Using a two-handed weapon (and unless you are a cleric of Grumush, chances are you aren't) is good, but you are limited to (long)spear or quarterstaff. Mace is what I usually see. And most DMs put a restriction on moving a two-hander back and forth for spellcasting that ends up being the similar to moving a one-hander to your light-shield-wearing hand. So you get disarm protection, but lower AC, and not much improvement on your weapon. And this is really getting off-topic. The point of the discussion isn't that monks are the best at tactical maneuvers- they just get the basic feats for free without the required Int and Combat Expertise. If they want to be better, they need to spend another feat on them, or get a Disarming Quarterstaff/Kama. It is an option, and handy when it comes up. As for grappling, they have a solid start on becoming a good one if they choose.

AtomicKitKat
2007-03-18, 08:37 PM
Really, the solution to it would be to give Monks options. In the same way that Warlocks get Least, Lesser, Greater and Dark Invocations, and Wu Jen have their elemental schools, split Monks up into different schools, with each school granting a different(albeit thematic for that school) Bonus Feat at different levels. There's so many to choose from. Animal, Soft, Hard, Defensive, Offensive, Adhes-oops, wrong -ive. Anyways, you get the idea. I might post a mock-up in the Homebrew later, if anyone's interested, to see how people feel about it. Will take a few hours though.

Rigeld2
2007-03-19, 12:22 AM
Using a two-handed weapon (and unless you are a cleric of Grumush, chances are you aren't) is good, but you are limited to (long)spear or quarterstaff.
Wrong.


One-Handed

A one-handed weapon can be used in either the primary hand or the off hand. Add the wielder’s Strength bonus to damage rolls for melee attacks with a one-handed weapon if it’s used in the primary hand, or ½ his or her Strength bonus if it’s used in the off hand. If a one-handed weapon is wielded with two hands during melee combat, add 1½ times the character’s Strength bonus to damage rolls.
Feel free to wield that Mace/Morning Star/Flail/whatever 2 handed.

Clementx
2007-03-19, 01:35 PM
Wrong. Feel free to wield that Mace/Morning Star/Flail/whatever 2 handed.
Which is not as good as wielding an actual two-hander, since you only gain 1/2 your Str more (and getting 14 Str in the first place is not the highest priority for a cleric) and your damage dice are still 1d6. Yeesh, over-literal much?

Rigeld2
2007-03-19, 05:29 PM
You mean 1d8 damage dice. Heavy Maces and Morning Stars do 1d8... Clerics arent proficient with Flails tho (theyre a martial weapon, not simple) so I was wrong there.

I'm not over-literal, I just like all sides of a debate to actually debate facts instead of try and state what they think is fact.

Raum
2007-03-19, 05:49 PM
Which is not as good as wielding an actual two-hander, since you only gain 1/2 your Str more (and getting 14 Str in the first place is not the highest priority for a cleric) and your damage dice are still 1d6. Yeesh, over-literal much?It's all of 1-2 points less than an average two hand weapon. The cleric can easily boost his strength with spells so he's not much behind a fighter, if any. More importantly, the 2 * Power Attack damage from two handing a weapon makes a bigger difference than the weapon's damage die. There's only a 1 point average difference between a 1d8 and a 1d10 after all.

the_tick_rules
2007-03-19, 05:54 PM
as with any class it's really how well you know how to use it's abilities that determines your success.

Illiterate Scribe
2007-03-19, 06:03 PM
and having your holy symbol on them is not RAW.

Aha! But what if your holy symbol is your shield?

Raum
2007-03-19, 06:30 PM
You'll still need a free hand for any spell with a somatic component.

Clementx
2007-03-20, 10:30 AM
Holy symbols are listed separately as an adventuring tool. It is a wooden or gold symbol of your god that fits in one hand. If you want to allow clerics to make their shields their focus, you are also ignoring the rules for spell components. You might as well let bat guano give a shield bonus, too.

Kaerou
2007-03-20, 11:22 AM
My main problem with Monks isnt their class abilities (yeah, from this thread pointing out, those are harsh anyway)

My main problem is that they all have to be lawful monestary type folk.

I want my martial artist who was trained normally / by an old master somewhere / whatever who can be neutral or chaotic, who doesnt go jumping through dimention doors etc.

Tobrian
2007-03-20, 11:26 AM
There are certainly a few points where the monk's abilities aren't as powerful as they seem at first glance but that's the case with every PHB core class except druid. Compared to later classes from non-core supplements they all feel underpowered. *shrug*


As yuki already said a decent wizard is both flying and doesn't give the monk a save.

So? A flying oponent is equally a problem for other classes, i.e. for a melee-fighter, or for a specialist wizard without ranged evocation spells. Heck, even an arcane archer is useless against a flying wizard if the wizard was clever enough to cast stuff like protection from arrows, sphere of invulnerability and stoneskin.

So then the monk buys a pair of boots of flying. End of problem.


That's why you have a locked gauntlet as a cleric and are a dwarf.

...because all clerics are dwarves, sure. And "decent" wizards only cast Fly and spells without saves. Do you always create your characters only with an eye towards optimizing against every possible combat event? You should play that new computer game, D&D Tactics, that WotC brought out. PvP all the time, no annoying roleplaying.


Not true, the wizard flies away and the cleric has plenty of options to defend against the monk.

List some.

Telonius
2007-03-20, 11:50 AM
Okay, yes, Wizards are the win. Even if fighting a Monk10/Chuck Norris10, the wizard will be able to summon a Greater Oar of Norrisbane and whack him into next Thursday before it's his turn. This thread isn't about wizards.

Back to the Monk.

I think the problems that the Monk has could largely be mitigated by giving him full BAB, without making it unbalanced to the rest of the melee classes.

Being pretty much the epitome of MAD, the Monk has to spend much more gold throughout his career to keep up all the ability scores than, for example, an equivalent Fighter. Monks typically sacrifice Strength, since their signature ability (flurry of blows) supposedly depends on hitting more often, rather than harder. Bumping up the BAB would help the ability actually mean something at lower levels. It'd also help in some of the Monk's other typically Monk-y stuff - tripping, disarming, and grappling.

By the way, has anybody ever seen a damage table comparing an unarmed monk's flurry, and a high-strength monk flurrying and power attacking 2h with a Q'staff? I haven't, but I'd be curious to see the results for a level 11 monk (Greater Flurry applies). It'd definitely be a nonstandard monk, probably having a low Wis instead of a low Str, and Power Attack instead of Weapon Finesse. (Maybe this guy is the half-orc monk that the PHB was talking about?)

Ramza00
2007-03-20, 12:19 PM
I think the problems that the Monk has could largely be mitigated by giving him full BAB, without making it unbalanced to the rest of the melee classes.

Full Bab, easy ways to enchant his fists (cheapily), piercing DR, and possibly some more synergistic abilities.

Kaerou
2007-03-20, 02:31 PM
I would liek to see the monk gaining his unarmed damage with his weapons..

Ive always -really- wanted to play a quarterstaff wielding martial artist.. of course, its just sub-par in D&D.

elliott20
2007-03-20, 02:31 PM
the monk's problem is that it's just a very poorly conceived concept.

It tries to do a mish-mash of all wuxia kungfu themes and mixes all of it into ONE package. the end result is just this class that looks and feels like an incredible niche class that might as well speak out of sync the entire time.

the swordsage is everything the monk was meant to be.

NemoUtopia
2007-03-20, 05:04 PM
I would liek to see the monk gaining his unarmed damage with his weapons..

Ive always -really- wanted to play a quarterstaff wielding martial artist.. of course, its just sub-par in D&D.

A staff wielding strict monk is, sure. But a Monk/Ftr is arguably not (actually, I'd argue that a Mnk/Ftr is better at strict HtH too, but that's neither here nor there. Sure, you lose some kung-fu trickery, but frankly, a monk with enough splash in fighter to get Weapon Specialization, actual combat feats, and a slightly better BAB is a better combat monk (note: takes a slight hit to Ref/Will saves). I realize that's not exactly the point of the thread, but to directly dove-tail to the topic: I just think many Monk abilities are superfluous for what a Monk should be as a class. Giving up some of these abilities, slowing the speed boost and slow fall progressions somewhat, and other minor tweaks should (in theory) allow a balanced monk variant with full BAB and some extra bonus feats.

Nowhere Girl
2007-03-20, 07:06 PM
Personally, I'd just like to know where these wizards are getting all of those spell slots from. The wizard I'm familiar with can prepare a total of four of each spell per level (five with specialization) at 20th level, but these wizards who have everything including the kitchen sink prepared at all times must have something like 10503284 slots per spell level for their spells. Having played spellcasters before and dealt with the reality of not having infinite spell slots, I'm jealous. :smalltongue:

Also, the wizards I know of can generally only cast one spell per round, not 50 or 60. And come to think of it, I don't know how that wizard was able to cast fly (standard action) and then ascend 100 feet in a single round, having first been guaranteed to win initiative despite focusing on intelligence over dexterity (I would think ...?). I must not have learned the secrets of uber wizardry. :smalltongue:

Maybe I just didn't know that all wizards are supposed to be designed from the ground up as Diemonk McMonkbane. Only I was under the impression from the last time people started throwing around 10,000 "perfect spells for the situation" that all spellcasters were designed from the ground up as Illfindyouscout the Scoutslayer.

I'm so confused. :smallfrown:

Ramza00
2007-03-20, 07:16 PM
Nowhere Girl (http://www.giantitp.com/forums/member.php?u=23835)
....
about wizards and overexageration
....Overland flight so you don't waste a round buffing casting flight. You can now use that action to move, buff something else, or attack.

And wizards have more like 6 spells per spell level (4 base, 1 specialist, 1 bonus spell) eventually this grows to about 7 per spell level as you level up (at 32 int, which you should have about lvl 20 you have 7 spells per spell lvl with all lvls besides 9th and 8th which with you have 6 spells, and 1-3rd where you have 8 spells you need 16 int at lvl 1 to get 32 int at lvl 18)

I still agree with you generally but not in the specific details.

1337_master
2007-03-20, 07:16 PM
Ya know, I remember a Long quote by one Mr. Red mage

Let’s look at it like this: A 20th level barbarian can Rage a couple more times, a 2oth level Rogue is really good at finding traps, and a 20th level Fighter has a hell of a lot of hit points and feats. A 20th level monk becomes an extra planar creature and is immune to a vast array of spells, effects, some physical damage, and has a lot of hit points, and can probably find a lot of traps, and can do more damage with his hands than just about anything in the game, dual wield with no problems, and got improved unarmed attack at first level. Yeah, the monk is just a bit unbalanced. Why do you think Black Belt's always helping us out?

Red Mage

Bears With Lasers
2007-03-20, 07:21 PM
Personally, I'd just like to know where these wizards are getting all of those spell slots from. The wizard I'm familiar with can prepare a total of four of each spell per level (five with specialization) at 20th level, but these wizards who have everything including the kitchen sink prepared at all times must have something like 10503284 slots per spell level for their spells. Having played spellcasters before and dealt with the reality of not having infinite spell slots, I'm jealous. :smalltongue:
The wizard you're familiar with isn't getting bonus spell slots from a high INT, apparently.
Believe me, a high-level spellcaster has MORE than enough spell slots for four or five encounters a day.


Also, the wizards I know of can generally only cast one spell per round, not 50 or 60. And come to think of it, I don't know how that wizard was able to cast fly (standard action) and then ascend 100 feet in a single round, having first been guaranteed to win initiative despite focusing on intelligence over dexterity (I would think ...?). I must not have learned the secrets of uber wizardry. :smalltongue:Overland Flight lasts all day. And keeps you safe from most things on the ground whether they win initiative or not.


Maybe I just didn't know that all wizards are supposed to be designed from the ground up as Diemonk McMonkbane. Only I was under the impression from the last time people started throwing around 10,000 "perfect spells for the situation" that all spellcasters were designed from the ground up as Illfindyouscout the Scoutslayer.

I'm so confused. :smallfrown:Um, they don't have to be. Wizards can disable monks with spells they'd have memorized anyway, becuase they can also disable fifty other things with those spells.

Stephen_E
2007-03-20, 07:53 PM
Regarding Enchanting Monk fists, it would help if they could focus their Ki Strike

Say every 3 levels after 4th (where they get Ki Strike (their fist become magic) each morning the Monk may focus his Ki to give them a +1 value enhancement bonus for the next 24 hours. The strength of the bonus increases by +1 every 3 levels after 7th. They can apply several lesser enhancements upto the max.

Example -
A 10th level Monk focuses his Ki to give his fists +1 enhancement, + Bane Orcs (he's planning on attacking an Orc encampment) for a total of +2 Enhancements.

Basically this is like the Artificer Personal Augumentation spell, but applied to the Monks unarmed attack, and only chosen once per day.

If you want more you could do something similiar with a armour enhancemnt. You could link the enhancement value to the Monk AC bonus, so at 5th level you get a +1 AC bonus as currently AND in addition get a +1 enhancement bonus, that could be a simple +1 AC or something like Light Fortification (I remember a campaign where everyone had either armour, shields or class ability that gave immunity to crits except for the poor Monk).

Stephen

The_Werebear
2007-03-21, 10:46 AM
Ok,

Give them Full BaB

Remove Flurry of Blows

Give them full skirmish progression.

Remove their Dimension Door

Give them the ability to make balance/jump checks to run on air a la Air Walk.

Allow them to take spring attack as a bonus feat

Possibly give them a Dervish Dance like ability.

As they gain more subtypes of their fists as weapons, start making them acutual enchanted weapons.

Tola
2007-03-21, 12:07 PM
Give them full skirmish progression.

Excuse me? Can you explain this? What is 'Skirmish', and why does it apply?

Piccamo
2007-03-21, 12:12 PM
Give them full skirmish progression.

Excuse me? Can you explain this? What is 'Skirmish', and why does it apply?

Skirmish is extra Precision Damage and Dodge AC gained after moving 10 feet. It is found in Complete Adventurer. It is a feature of the Scout class. It applies because, like the scout, the monk is intended to be a mobile fighter.

Telonius
2007-03-21, 12:24 PM
It's an ability from the Scout class - similar to Sneak Attack, though you have to move to get it.

I'm not sure that it really fits the Monk flavor, though if the Monk manages to actually get an attack off against the spellcaster it helps with what the monk is supposed to do mechanically. The "Cloud Walk" idea is about the same... helps him get up to the flying Wizard. You'd really have to work to get that fluff to make sense, and it'd make the Monk an even weirder mix of powers than he is now.

The_Werebear
2007-03-21, 01:22 PM
It's an ability from the Scout class - similar to Sneak Attack, though you have to move to get it.

I'm not sure that it really fits the Monk flavor, though if the Monk manages to actually get an attack off against the spellcaster it helps with what the monk is supposed to do mechanically. The "Cloud Walk" idea is about the same... helps him get up to the flying Wizard. You'd really have to work to get that fluff to make sense, and it'd make the Monk an even weirder mix of powers than he is now.

The monk class is composed largely from a variety of Martial arts flicks. How many of those allow the martial artists to make amazing leaps that end with running on air almost.

As for skirmish, I think it fits with the idea of doing a running kick to the face. More momentum from the attack, you are harder to hit, and it fits the idea of an agile combatant charging foward, hitting you, then rolling past before you can counter.

elliott20
2007-03-21, 03:51 PM
the problem is, not all martial artists are the same. Hell, not even all the agile, kungfu fighters are the same. But the monk class that is presented here has neatly pigeonholed every would be unarmed martial artist into becoming a high jumping, unarmored, unarmed multi-attack zen fighter, who runs lightning fast but can't hit crap.

as such, the monk class just isn't a very good class to begin with.

Hell, even the warrior monks from all those martial art flicks were not like this. They probably had more in common with a fighter then they did with the monk presented in the PHB.

Talya
2007-03-21, 03:59 PM
Mage: I take a 5 foot step away from him, and cast Wall of X/Fly/Invisibility/Disintegrate.


Disintegrate is a touch attack. Bad choice against a monk.

Epiphanis
2007-03-21, 11:51 PM
Consider:

1) Terrain. The Fly spell and long-range attack spells are not so incredibly advantageous to a wizard encountering a monk in a 30' x 30' dungeon room with a 10' high ceiling. Conversely, in a wide-open space the monk can usually circumvent a wizard's defending allies with superior movement and tumbling.

With opposing parties in an open space, each with spellcasters to neutralize each others' defenses, a side with a monk instead of a fighter probably has an advantage over one with a fighter instead of a monk -- the fighter's wizard will probably be the first casualty, and the rest of his party will likely follow.

2) Initiative. In a one-on-one fight between wizard and monk, the crucial factor is who goes first and if they can land their first strike. A monk who lands a stunning blow before the wizard can cast his first spell is probably going to win. On the other hand, a wizard with the drop can get the upper hand with the right choice of spell.

3) Preparation. Monks are less versatile than wizards, but are pretty much prepared to do everything they can do at any point in time. Wizards have to anticipate what they will need. A wizard busting into the Monastery of the Master of the North Wind will probably be appropriately armed, but a random encounter is likely a good deal less so.

Aquillion
2007-03-22, 02:35 AM
...what is with this obsession with one-on-one class-vs-class fights lately? D&D isn't designed with that in mind.

In terms of actual game balance, I think most people can agree that monks have serious problems. They don't offer any support to anyone else. They're fairly weak attackers for a melee-focused class (which means they're poor grapplers, in the end.) Their enhanced movement is nice, but at the end of the day they'll have to depend on magical items to get flight like every other non-caster that doesn't get it naturally. They get few useful abilities outside of combat, and the ones they do get are far below even partial casters. Their saves don't help the party very much--a low save can be a problem for an otherwise useful class (e.g. fighters and will saves), but good saves on their own don't do anything.

Now, there are fixes for some of these, but I think there's a much more basic problem here: Monks don't belong in core. Period. Almost all the other core classes represent some general archetype, a versatile outline that represents many different characters from multiple fantasy genres. More to the point, they represent basic roles from those genres, roles that can be easily adapted to almost any setting and almost any adventure. Because they draw on so many sources, wizards and fighters and rogues and bards and rangers and clerics and whatever all have things they can do and contribute under most situations. Paladins can be easily adapted into any sort of holy warrior; druids (although they're a little specific, too) can be adapted into any sort of nature-based hermit-mage.

Monks... don't work like that. They're from a specific type of Hong Kong action movie (and a few similar things); they have no place too far outside it, and it shows. The reason why their abilities seem so bizarrely assigned is because the people making the class knew perfectly well that monks were too specific to fit into Core, and hadn't the faintest idea what to do with them.

Sure, you could make a theoretical fix for monks by just pouring new powers into them and giving them everything a melee class needs or arbitrarily decide that they're going to be mage-killers or whatever, but nothing can give them a true place in Core, because they don't have one. Having monks in Core is like having Warforged or Samurai or Wu-Jen in Core--it's absurd on the face of it, and their hopelessly disorganized and mismatched abilities are only a symptom of this. With a little work, monks could be an extremely fun to play as one of the characteristic Oriental Adventures-specific classes, but trying to shoehorn them into a core role is simply never going to work.

Yes, I know that this fix is never really going to be implemented by anyone, but it's the long and short of it. Trying to give monks a clearly-defined role in the D&D metagame is a waste of time when you can't even give them a role thematically.

Jack Mann
2007-03-22, 02:42 AM
Monks are often sold as wizard-killers. I think it's been adequately demonstrated that they aren't. But even if they were, that still wouldn't help them overmuch. As Aquillion said, this isn't about PvP. While players may face off against casters every now and again, that's still only a few fights in which the monk would be contributing. That's poor design. And as we've seen, they can't even contribute then.

martyboy74
2007-03-22, 06:37 AM
Disintegrate is a touch attack. Bad choice against a monk.
...that would be against the ground beneath them (in hindsight, probably chained), to create a pit for which they have no save to get out of.

Also, this's kind of late.

Jack Mann
2007-03-22, 06:48 AM
Sadly, chained disintegrate probably wouldn't work. You wouldn't have line of sight to those deeper cubes until the first one was destroyed.

It's wonderful against walls, though.

Orzel
2007-03-22, 08:40 AM
My own homebrew of the monk turns them into a rogue with better defense and more consistently with damage. They lose Flurry of Blows, Diamond Body, Diamond Soul, Tongue blah blah, and Timeless Body. Instead their unarmed strike increases one size and gains additional dice as they level (1 more every 4th level) so a 20th level medium monk deals 6d4 damage. They also lose Abundant step but can meditate a full round to learn a 4th or lower wizard spell once per day and cast it every 4 rounds.

They basically are rogues with more reliable offense and defense but worse utility and potential.

Sir Giacomo
2007-03-22, 09:23 AM
Hmmm, have not read the entire thread, only the last dozen or so posts.

Further up I posted that I believe the monk to be the relatively weakest class, even in combat vs casters the fighters seem to be ahead with comp.longbow attacks (plus the specialisation options and feats here), as well as with simply taking the improvedunarmedstrike/stunningfist feat themselves and hitting with 5 better BAB.

However, since I'm currently trying to revise a high-level fighter core character, I have come to notice that there are two key abilities in the game, most important at high level combt: DEX and WIS. And the monk makes great use out of these.

Now, there are two major attacks of the monk that do not need strength and are based on DEX (if you use weapon finesse feat) to hit and on WIS for damage and can quite quickly become a nuisance for spellcasters: quivering palm and stunning fist. You can use both in the same round. Both necessitate a fort save.
Now that save, for a monk at level 20, is 10+10 (half his level) + his WIS modifier.
If a monk maxes out WIS with age (a nice byproduct of his timeless body ability, something only a druid can do as another class), then he could with magic items, plus four stat gains (and a starting WIS of 16) well get to a WIS of 34, or a bonus of +12. Having gloves of DEX +6 as well and a starting value of 16, plus some more inherent boni and the remaining stat gain, the DEX bonus could also approach +8, which with an improved initiative feat is getting the monk quite high up the initiative ladder.
This sets the DC for the stunning fist and quivering palm to 32. There may be even non-core items out there that increase it.*

Now, a spellcaster needs to resort to morphing spells to even have a chance to get the fortitude saves up to escape these attacks (and not that, for instance, a cloak of resistance no longer fits most of those fort-strong morphs; a good exception is a barbed devil, courtesy greenknight). One of the worst CR 20 creatures in the game, a balor (may also be considered a spellcaster), has a fort save of +22, making it quite likely that it will fail one of this saves when hit by a monk's full attack.

Add to this the high move, a jump quickly getting to the height of 30ft that a spellcaster can go directly up after casting the fly spell, plus the dimension door abilities, and you have quite a threat for spellcasters, actually the worst non-spellcasting threat there is to them. Ah, and to get a composite bow for hitting at range, you can choose to be an elf monk (or use a feat to get the bow).
Grapple tactics for a monk a likely not a good idea since at high level, almost all casters should have access to freedom of movement effects (even the monk should get it through a ring if he goes the DEX/WIS route to avoid opponent babarians crushing him).

- Giacomo

Edit: *I remember somewhere in the spell compendium there is a fairly low-level divine spell that gives something like +10 sacred bonus to WIS. If that could be turned into an item, or into a potion, the monk would rock even more

Aquillion
2007-03-22, 12:31 PM
They also lose Abundant step but can meditate a full round to learn a 4th or lower wizard spell once per day and cast it every 4 rounds.Er, I don't think I get what you're saying here. So they mediate a single round and learn one spell a day (to be clear here--any wizard spell with one round of preparation, though I assume they only get to keep it for that day), and then they can then cast it every 4 rounds for the rest of the day? I assume I'm misunderstanding something here. Casting Greater Invisibility or Wall of Ice or Black Tentacles or Dimension Door, or um, Polymorph (which would be what everyone would always choose) once every four rounds would get nasty fast.

Piccamo
2007-03-22, 12:34 PM
He says 4th or lower wizard spell so its not ANY wizard spell.

Orzel
2007-03-22, 04:30 PM
Er, I don't think I get what you're saying here. So they mediate a single round and learn one spell a day (to be clear here--any wizard spell with one round of preparation, though I assume they only get to keep it for that day), and then they can then cast it every 4 rounds for the rest of the day? I assume I'm misunderstanding something here. Casting Greater Invisibility or Wall of Ice or Black Tentacles or Dimension Door, or um, Polymorph (which would be what everyone would always choose) once every four rounds would get nasty fast.

The Caster level is terrible (1/4), requires a cumulative Concentration check to meditate, and requires a high enough Int score. Few monks have 18 Int and max Concentration. Abusing it requires excellent rolls or self-nerfing.